If a force of 12 N is applied to an object
and it accelerates 4 m/s2, what is the
mass of the object?

Answers

Answer 1

Answer:

The answer is 3 kg

Explanation:

The mass of the object can be found by using the formula

[tex]m = \frac{f}{a} \\ [/tex]

f is the force

a is the acceleration

From the question we have

[tex]m = \frac{12}{4} \\ [/tex]

We have the final answer as

3 kg

Hope this helps you


Related Questions

HELP ASAP!!!
A force of 50N acts in an easterly direction on an object at the same time as a force of 80N pushes on it in the direction of N 45 degrees E. Determine the magnitude and direction of resultant force

Answers

Write each force in component form:

v ₁ : 50 N due east   →   (50 N) i

v ₂ : 80 N at N 45° E   →   (80 N) (cos(45°) i + sin(45°) j ) ≈ (56.5 N) (i + j )

The resultant force is the sum of these two vectors:

r = v ₁ + v ₂ ≈ (106.5 N) i + (56.5 N) j

Its magnitude is

|| r || = √[(106.5 N)² + (56.5 N)²] ≈ 121 N

and has direction θ such that

tan(θ) = (56.5 N) / (106.5 N)   →   θ ≈ 28.0°

i.e. a direction of about E 28.0° N. (Just to clear up any confusion, I mean 28.0° north of east, or 28.0° relative to the positive x-axis.)

If it takes 100 N to move a box 5 meters, what is the work done on the box?

A. 20 J
B. 5,000 J
C. 5 J
D. 500 J​

Answers

Answer:  D.   500 J

=========================================================

Explanation:

To find the amount of work done, we multiply the force by displacement

work = force*displacement

work = (100 N)*(5 m)

work = (100*5) Nm

work = 500 J

In this case, "Nm" refers to "Newton meters" and not "nanometers"

1 newton meter is equal to 1 joule

fill in the word that completes the statement. the direction of the magnetic field produced by an electric current will change when the direction of the _____ changes​

Answers

Answer:

current trust me thoo pls

Answer:

Current

Explanation:

Right on edge ✅

A rocket has a mass of 142 kg. If you want to give it an acceleration of 7 m/s2 how much force does the engine need to apply

Answers

994 N

Explanation:

The force acting on an object given it's mass and acceleration can be found by using the formula

force = mass × acceleration

From the question we have

force = 142 × 7

We have the final answer as

994 N

Hace this helps you

Reena has rolled down her toy car through a ramp. When she increases the height of the ramp (10 cm ) the car has travelled 5 metres and she increases the height of the ramp (13 cm) the car has travelled 6 metres . What happened when she raises the height of the ramp?

Answers

Answer:

As she raises the height of the ramp, the speed of the toy car decreases.

Explanation:

Speed is the ratio of the distance traveled by an object to the time taken.

i.e speed = [tex]\frac{distance}{time}[/tex]

From the given question, it would be observed that:

i. when the height of the ramp was 10 cm, the car traveled 5 meters.

ii. when the height of the ramp was increased to 13 cm, the car traveled 6 meters.

Therefore, an increase in the height of the ramp causes a decrease in the speed of the toy car so that the distance traveled per unit time increases. Showing that there is an inverse relationship between the height of the ramp and the speed of the toy car.

Un gas ideal ocupa un volumen de 4.00 m3 a una presión absoluta de 200 kPa. ¿Cuál será la nueva presión si el gas es comprimido lentamente hasta 2.00 m3 a temperatura constante?

Answers

Given :

Initial pressure, [tex]P_i=200\ kPa[/tex].

Initial volume, [tex]V_i= 4\ m^3[/tex].

Final volume, [tex]V_f = 2 \ m^3[/tex].

To Find :

Final pressure, [tex]P_f[/tex].

Solution :

We know, at constant temperature :

[tex]P_iV_i=P_fV_f\\\\P_f=\dfrac{P_iV_i}{V_f}\\\\P_f=\dfrac{200\times 4}{2}\\\\P_f=400\ kPa[/tex]

Therefore, new pressure is 400 kPa.

Hence, this is the required solution.

who songs are better jason derulo or Dababy , roddy ricch

Answers

Answer:

roddy i guess?

Explanation:

Answer:

Dababy and Roddy Ricch. Jason's songs use to be fire years back but now....not so much.

Explanation:

There is a ball at rest on a flat floor. What kind of equilibrium is it in? *

Stable Equilibrium
Unstable Equilibrium
Neutral Equlibirum
Not in Equilibrium

Answers

answer : neutral equilibrium

Marla made ice by freezing liquid water the freezerWhat did the freezer do to cause the change in the water?

Answers

Answer:

It slowed the kinetic energy of the water.

Explanation:

When you freeze something, the cold air slows down the kinetic energy of that object. There is no actual "cold" but there *is* a lack of heat, and this is proven in that example. Since temperature is a measure of how much kinetic energy an object has, the lack of kinetic energy would make the objects colder. Therefore, when you put a bottle of water into the freezer, by slowing the kinetic energy of the water, it freezes it!

Hope this helps <3

Convert 100°c into Faherenite and kelvin in steps.​

Answers

Answer:

100°c = 373.15 K

100°C=212°F

Explanation:

Given the 100°C

To convert Celsius to Kelvin, we need the following equation.

°C + 273.15 = K

100°C + 273.15 = K

373.15 = K

Therefore, 100°c = 373.15 K

Celsius To Fahrenhite:

F = 9/5C + 32

   =9/5(100)+32

  = (180) + 32

  = 212°

Therefore,

100°C=212°F

What is the specific heat of a substance that absorbs 2500 joules of heat when a sample of 100 g of the substance increases in temperature from 10 degrees Celsius to 70 degrees Celsius?

Answers

Answer:

I'm sorry to inform that I couldn't find an answer to this question. Socratic can help! This learning app, powered by Google AI, helps you understand your school work at a high school and university level. Ask Socratic a question and the app will find the best online resources for you to learn the concepts.

Explanation:

Which of the following is a FALSE statement? (A) Heat moves through solids by conduction (B) Molecules move faster in warmer substances (C) Warm water is denser than cold water (D) Heat moves through liquids and gases by convection

Answers

i believe c is a false statement because the warmer something is the faster the molecules move

Answer:

C.

Explanation:

QUICKK I NEED HELP!!
How is a mineral different from an element rock

Answers

Answer:here you goo

Explanation:

A mineral is a naturally occurring inorganic element or compound having an orderly internal structure and characteristic chemical composition, crystal form, and physical properties. ... A rock is an aggregate of one or more minerals, or a body of undifferentiated mineral matter.

Answer:

Minerals are naturally occuring solid while rocks are naturally occuring substances consist of aggregate minerals clumped together with other materials

A 200 N net force acts on a 50 kg box. What is the acceleration of the box?

Answers

Answer:

a= 4m/s^2

Explanation:

Fnet =200N

m=50kg

a=?

Fnet = ma

200=50a then divide both sides by 50 to get acceleration

Taking into account the Newton's second law, the acceleration of the box is 4 [tex]\frac{m}{s^{2} }[/tex].

In first place, you have to know that Newton's second law, called the fundamental law or fundamental principle of dynamics, states that a body accelerates if a force is applied to it.

Then, the amount of force necessary is proportional to the acceleration that the body acquires, and the constant of proportionality  is the mass of the body.

Mathematically, Newton's second law is:

F= m×a

where:

F = Force [N] m = Mass [kg] a = Acceleration [m/s²]

In this case, you know:

F= 200 N m= 50 kg a= ?

Replacing in the Newton's second law:

200 N= 50 kg×a

Solving:

a=200 N÷ 50 kg

a= 4 [tex]\frac{m}{s^{2} }[/tex]

In summary, the acceleration of the box is 4 [tex]\frac{m}{s^{2} }[/tex].

Learn more about the Newton's second law:

brainly.com/question/23845187?referrer=searchResults brainly.com/question/13959891?referrer=searchResults

In which scenario will the two objects have the least gravitational force between them?​

Answers

Answer:

A

Explanation:

In  scenario A the two objects have the least gravitational force between them.

What is  gravitational force?

Gravitational force is defined as the force which one mass exert on the other mass separated by finite distance. Mathematically gravitational force is directly proportional to product of mass and inversely proportional to square of distance between them.

Force between two mass,

F = Gmm/r²

Given question, to find when have the least gravitational force between two masses so from option A we see there separation distance is maximum 1.5 m and mass product is less than option B.

So least gravitational force when masses are 12 kg and separation distance is 1.5 meter.

To learn more about gravitational force refer to the link:

brainly.com/question/12528243

 #SPJ5

Which of the following actions will increase the inertia of a shopping cart?
-Adding items to the cart
-
-Removing the wheels from the cart
O Rearranging items in the cart

Oiling the cart's wheels

Answers

Answer:

A. Adding items to the cart

Explanation:

Inertia can be defined as the tendency of an object or a body to continue in its state of motion or remain at rest unless acted upon by an external force.

In physics, Sir Isaac Newton's first law of motion is known as law of inertia and it states that, an object or a physical body in motion will continue in its state of motion at continuous velocity (the same speed and direction) or, if at rest, will remain at rest unless acted upon by an external force.

The inertia of an object such as a shopping cart is greatly dependent or influenced by its mass; the higher quantity of matter in a shopping cart, the greater will be its tendency to continuously remain at rest.

Hence, adding items to the cart is an action which will increase the inertia of a shopping cart, by increasing its tendency to be at rest.

Adding items to the cart will increase the inertia of a shopping cart.

Inertia refers to the resistance that an object has to modify its velocity and/or direction during movement.

Inertia not only includes the speed of the object but also the direction of its movement.

Inertia also can be described as the inherent tendency of a given object to resist changes in its state of movement (motion).

In conclusion, adding items to the cart will increase the inertia of a shopping cart.

Learn more in:

https://brainly.com/question/3268780

4. A box of books weighing 325 N moves at a constant velocity across the floor when the box is pushed with a force of 425 N exerted downward at an angle of 35.2° below the horizontal. Find mk between the box and the floor.

Answers

Answer:

0.61°

Explanation:

Since the box move at constant velocity, it means there is no acceleration then we can say it has a balanced force system.

Pulling force= resistance force

From the formula for pulling force,

F(x)= Fcos(θ)

= 425×cos(35.2)

=347N

The force exerted downward at an angle of 35.2° below the horizontal= Fsin(θ)= 425sin(35.2)

=425×0.567=245N

Resistance force= (325N+ 245N) (α)= 570N(α)

We can now equates the pulling force to resistance force

570 (α)= 347N

(α)= 347/570

= 0.61

which is more important while hitting a home run a heavier bat or faster swing? why? ​

Answers

Answer:

Explanation:

the hitting should be in the almost top so the more faster and strong u hit more the ball more at top will go and more far will go

if you walk at an avaerage speed of 5 km/h for 30 min how far will you walk

Answers

Answer:

2.5 km/30min

Explanation:

Divide 5 by 2 because 30  min is half of 1 hour. when you do that you get 2.5

A rocket is launched from the origin with an acceleration of 20.0 m/s2 in a straight line at an angle of 30.0 degrees above the horizontal. The launch acceleration lasts for 2.00 seconds at which time the fuel is exhausted. The rocket then falls with an acceleration of 9.80 m/s2 downward. What is the time it takes to reach maximum height?

Answers

Answer:

4.04 seconds

Explanation:

Context before solving:

In order to solve this problem, we must keep in mind that the initial 2.00 seconds at which the fuel is exhausted does not signal when the rocket reaches its maximum height.

From this moment on, the rocket has a downward acceleration of 9.8 m/s² but it still has an upwards velocity, which we will calculate. This upwards velocity keeps the rocket moving up for a certain period of time, which we will also calculate.

For this problem, let's set the upwards direction to be positive and the downwards direction to be negative.

To find the time that the rocket takes to reach its maximum height, we are going to use the initial 2.00 seconds and find the additional seconds it takes to reach a final vertical velocity of 0 m/s. This represents the time at which the rocket stops moving and heads in the downwards direction, which takes place after the rocket has reached its maximum height.

Solving for initial velocity:

The time in the air of an object in projectile motion can be found using this equation, derived from one of the constant acceleration kinematic equations.

Time in the air (projectile motion):

[tex]$t=\frac{2v_isin\theta}{g}[/tex] where g = gravitation acceleration = 9.8 m/s²

Solve for [tex]v_i[/tex] by plugging in 2.00 seconds for t, 30 degrees for theta, and 20.0 m/s² for acceleration, since this is not a constant acceleration problem.

[tex]$2.00=\frac{2v_isin(30)}{20.0}[/tex]

Multiply sin(30) and 2 together.

[tex]$2=\frac{v_i}{20}[/tex]

Multiply 20 to both sides of the equation.

[tex]$40=v_i[/tex] [tex]v_i=40[/tex]Finding the vertical component:

Now we know that the initial velocity of the rocket is 40 m/s. We need to solve for the vertical component of the rocket's velocity in order to solve for the additional time it took after the 2.00 seconds to reach its maximum height.

Vertical component:

[tex](v_i)_y=v_i \times sin\theta[/tex] [tex](v_i)_y=(40) \times sin(30)[/tex][tex](v_i)_y=20[/tex]

The vertical component of the velocity vector is 20 m/s.

Finding additional seconds after 2.00 s:

Now, in order to solve for the additional seconds that the rocket took to reach its maximum height, let's use one of the kinematic constant acceleration equations that uses the variables [tex]v_f[/tex], [tex]v_i[/tex], [tex]a[/tex], and [tex]t[/tex].

[tex]v_f=v_i + at[/tex]

Since we are trying to solve for time, we need to use this equation in terms of the vertical direction, aka the y-direction. Time is the same in either case.

[tex](v_f)_y=(v_i)_y+a_yt[/tex]

The final vertical velocity of this rocket is 0 m/s at the top, or its maximum height. We found that the vertical component, aka the rocket's initial vertical velocity, is 20 m/s. The acceleration is given to us: -9.8 m/s² (since it's falling downwards, the acceleration must be negative because we already established this in the beginning).

We are trying to solve for time t. Substitute the known values into the equation.

[tex]0=(20)+(-9.8)t[/tex]

Subtract 20 from both sides of the equation.

[tex]-20=-9.8t[/tex]

Divide both sides of the equation by -9.8.

[tex]2.040816327=t[/tex] [tex]t=2.04\ \text{seconds}[/tex] Finding total time to reach max height:

Now we can take this time and add it to the initial 2.00 seconds of the rocket. This time we just solved for is the time after these initial seconds that the rocket kept going upwards, since its initial vertical velocity was not 0 m/s yet.

[tex]2.00+2.04=4.04 \ \text{seconds}[/tex]

The time that the rocket takes to reach its maximum height is 4.04 seconds.

Answer:

4.04 second is your answer hope it help you........

The ___________________ ______________ (two words) is the least dense layer of the Earth.

Answers

Answer:

lithosphere

Explanation:

the least densest layer is the outer most layer

the densest layer is the inner most layer and the inner most layer is the inner core

Answer:

lithosphere

Explanation:

i took the test and i got it right

What is an electron?​

Answers

Answer:

A stable subatomic particle with a charge of negative electricity, found in all atoms and acting as the primary carrier of electricity in solids.

Explanation:

In more simple terms, an electron is a negatively charged subatomic particle.

Hope that helps :)

Answer:

An electron is a particle with negative charge that can be found in the atom, along with neutrons and protons

If the velocity of an object is zero, then that object cannot be accelerating. | True or False

Answers

Answer: False

Explanation:

A car travels 19 km in 30 minutes. What was its speed in km/hour?
HURRY WILL MARK BRAINLIST

Answers

19km in 30mins

1 hour is twice of 30 mins.

Twice the time means twice the distance.

therefore 2(19km) in 2(30 mins)

19km in 30mins = 38km/hour

please help lol
im also on my sisters acc btw

Answers

Answer:

D. Smooth muscle

Explanation:

Smooth muscle is a type of muscle tissue which is used by various systems to apply pressure to vessels and organs. Smooth muscle is composed of sheets or strands of smooth muscle cells. These cells have fibers of actin and myosin which run through the cell and are supported by a framework of other proteins.

Can I get brainliest please since I have enough explanation

A ball thrown vertically upward is caught by the thrower after 2.93 s. Find the initial velocity of the ball. The acceleration of gravity is 9.8 m/s 2 . Answer in units of m/s.

Answers

Answer:

The initial velocity of the ball is 28.714 m/s

Explanation:

Given;

time of flight of the ball, t = 2.93 s

acceleration due to gravity, g = 9.8 m/s²

initial velocity of the ball, u = ?

The initial velocity of the ball is given by;

v = u + (-g)t

where;

v is the final speed of the ball at the given time, = 0

g is negative because of upward motion

0 = u -gt

u = gt

u = (9.8 x 2.93)

u = 28.714 m/s

Therefore, the initial velocity of the ball is 28.714 m/s

An object O is acted upon by a 6 N force and a 5 N force. The angle between these two forces is 60°. Draw a scale diagram showing the forces acting on object O and determine the resultant force.​

Answers

Answer:

Resultant force = [tex]\sqrt{91}[/tex]N

Explanation:

Rf = [tex]\sqrt{f1²+f2²+2.f1.f2.cos60}[/tex]

Rf = [tex]\sqrt{6²+5²+2.6.5.1/2}[/tex]

Rf = [tex]\sqrt{36+25+30}[/tex]

Rf = [tex]\sqrt{91}[/tex]

doesn't look right though...

The magnitude of resultant force is 9.54 N and it is acting an angle 33° with the direction of 5 N force.

What is force?

An object's push or pull is seen as exerting a force. The interaction of the objects produces push and pull. You can also use words like stretch and squeeze to describe force.

The definition of force in physics is: The push or pull on a mass-containing item changes its velocity.

Given parameters:

Two forces are 6 N and 5N.

Angle between them is 60 degree.

So, magnitude of resultant force be = √(6² + 5² + 2× 6 × 5 × cos60°) N =  = 9.54 N.

Angle of the resultant force with respect to 5 N is = tan⁻¹{6sin60°/(5 +6 cos60°)  } = 33⁰.

Hence, magnitude of resultant force is 9.54 N and it is acting an angle 33° with the direction of 5 N force.

To find more about force, refer the link:

https://brainly.com/question/13191643

#SPJ5

Where do you think most of the water
will go?
split the same
though the path of little rocks
though the path of big rocka

Answers

Probably path of Little Rock’s because the big rocks won’t let water in :)

Can you give
an
example of a physical quantity
which cannot be measured?​

Answers

Answer:

Feeling's.

Explanation:

Nobody can measure the feeling of a person or love , etc

Foolishness cannot be measured

A linebacker strikes a fullback with full force in a football game.The reaction force to the force of the linebacker striking the fullback is ____.
a. the fullback falls backwards to the ground
b. the force of the fullback colliding with the ground
c. the fullback fumbles
d. the fullback applies a force to the linebacker
e. the linebacker applies a force to himself.
f. the fullback applies a force to himself
g. the sound of the pads colliding
h. there is a 3-yard loss and the linebacker is applauded

Answers

a. the back falls backwards to the ground
Other Questions
Pleassssseeee help. I neeed help About 50% of babies born with a certain ailment recover fully. A hospital is caring for six babies born with this ailment. The random variable represents the number of babies that recover fully. Decide whether the experiment is a binomial experiment If it is, identify a success, specify the values of n, p, and q, and list the possible values of the random variable x.(a) Is the experiment a binomial experiment? Yes No(b) What is a success in this experiment? This is not a binomial experiment. Baby recovers Baby doesn't recover(c) Specify the value of n.A. n = _____B. This is not a binomial experiment.(d) Specify the value of p.A. p = _____B. This is not a binomial experiment. Please help this is due in 30 mins The equation y = 13x represents the rate, in gallons per minute, that Tank A at an aquarium fills with water. The table represents the rate that Tank B fills with water. Determine which tank fills faster. PT2 To my last question!!!! please answer quickly What is the remainder when 19x^2 + 15x + 12 is divided by x - 2? PLSS HELPPPPPPPP HELPP ASP Maya will deposit $1,650 in an account that earns 5% simple interest every year. Her brother, James, will deposit $1,600 in an account that earns 8% interest compounded annually. The deposits will be made on the same day, and no additional money will be deposited or withdrawn from the accounts. Compare and contrast the two siblings' accounts after four years. What is the factored form of the equation 4x^2-11+6=0 Why are the coral reefs suffering? (site 2) explain Evaluate each value expression for the given valueof x: 20-3x, x=5 saved the colony of Jamestown from disaster through his leadership PLEASE HELP! THANK YOU!!! What is the subject in the following sentence?Didn't that old war movie end strangely? I am writing a paragraph and I need your help if you found any wrong grammar.Imagine going with your family on a filed trip in the vacation, and going back from it loosing one member. blaming yourself and feeling hapless for not being their or get to help that member Con quin compara Fernn Caballero el mar ? Por qu? Homer is 30 feet below sea level and Marge is 45 feet above sea level. Barts elevation is at the exact midpoint of those two values. What is the expression that model this problem, and which is Barts elevation? Read the excerpt from The Call of the Wild.They saw him marching out of camp, but they did not see the instant and terrible transformation which took place as soon as he was within the secrecy of the forest. He no longer marched. At once he became a thing of the wild, stealing along softly, cat-footed, a passing shadow that appeared and disappeared among the shadows. He knew how to take advantage of every cover, to crawl on his belly like a snake, and like a snake to leap and strike. He could take a ptarmigan from its nest, kill a rabbit as it slept, and snap in mid air the little chipmunks fleeing a second too late for the trees. Fish, in open pools, were not too quick for him; nor were beaver, mending their dams, too wary. He killed to eat, not from wantonness; but he preferred to eat what he killed himself. So a lurking humor ran through his deeds, and it was his delight to steal upon the squirrels, and, when he all but had them, to let them go, chattering in mortal fear to the treetops.Which theme does this passage illustrate?A good leader is strong and intelligent and treats his followers well.Only the strong survive in the wilderness.The desire to fight for power is an instinct.The only way to learn something is to try it. Both the North Cascades and southern Blue Ridge consist of crystalline rock and both are part of the Hinterland Deformation Belt structural province. Why are the North Cascades so much more rugged than the southern Blue Ridge? Be specific with respect to elevation, relief, precipitation, and glaciation. What is the difference between an expression and an any quality